K
Khách

Hãy nhập câu hỏi của bạn vào đây, nếu là tài khoản VIP, bạn sẽ được ưu tiên trả lời.

28 tháng 9 2020

Ta có: 

\(A=xy+yz+zx-x^2-y^2-z^2\)

\(\Rightarrow2A=2xy+2yz+2zx-2x^2-2y^2-2z^2\)

\(=-\left(x^2-2xy+y^2\right)-\left(y^2-2yz+z^2\right)-\left(z^2-2zx+x^2\right)\)

\(=-\left(x-y\right)^2-\left(y-z\right)^2-\left(z-x\right)^2\)

=> \(A=-\frac{\left(x-y\right)^2+\left(y-z\right)^2+\left(z-x\right)^2}{2}\le0\left(\forall x\right)\)

Dấu "=" xảy ra khi: \(\hept{\begin{cases}\left(x-y\right)^2=0\\\left(y-z\right)^2=0\\\left(z-x\right)^2=0\end{cases}}\Rightarrow x=y=z\)

Vậy Max(A) = 0 khi x = y = z

28 tháng 9 2020

Ta có A = xy + yz + zx - x2 - y2 - z2

=> 2A = 2xy + 2yz + 2zx - 2x2 - 2y2 - 2z2

=> 2A = -(x2 - 2xy +  y2) - (y2 - 2yz + z2) - (x2 - 2zx + z2)

=> 2A = -(x - y)2 - (y - z)2 - (z - x)2

=> 2A = -[(x - y)2 + (y - z)2 + (z  - x)2]

=> A = \(\frac{-1}{2}\left[\left(x-y\right)^2+\left(y-z\right)^2+\left(z-x^2\right)\right]\le0\forall;y;z\)

Dấu "=" xảy ra <=> \(\hept{\begin{cases}x-y=0\\y-z=0\\z-x=0\end{cases}}\Rightarrow x=y=z\)

Vậy Max A = 0 <=> x = y = z

11 tháng 10 2017

26 tháng 1 2019

Chọn đáp án C.

Vì x, y ,z > 0 nên x + y > 0; y + z > 0 và x + z > 0

Ta có:

Trắc nghiệm: Chương 1 Đại Số 9 (nâng cao) - Bài tập Toán lớp 9 chọn lọc có đáp án, lời giải chi tiết

Khi đó

A = x(y + z) + y(x + z) + z(x + y)

= xy + xz + xy + yz + xz + zy = 2(xy + yz + zx) = 2

24 tháng 12 2020

nhờ mn giúp mk bài này vs ạ

mk đang cần gấp !

cảm ơn mn nhiều

NV
25 tháng 12 2020

Đặt \(\left(\sqrt[3]{x};\sqrt[3]{y};\sqrt[3]{z}\right)=\left(a;b;c\right)\) \(\Rightarrow a^6+b^6+c^6=3\)

\(a^6+a^6+a^6+a^6+a^6+1\ge6a^5\)

Tương tự: \(5b^6+1\ge6b^5\) ; \(5c^6+1\ge6c^5\)

Cộng vế với vế: \(18=5\left(a^6+b^6+c^6\right)+3\ge6\left(a^5+b^5+c^5\right)\)

\(\Rightarrow3\ge a^5+b^6+b^5\)

BĐT cần chứng minh: \(\dfrac{a^3}{bc}+\dfrac{b^3}{ca}+\dfrac{c^3}{ab}\ge a^3b^3+b^3c^3+c^3a^3\) 

Ta có:

\(\dfrac{a^3}{bc}+\dfrac{b^3}{ca}+\dfrac{c^3}{ab}\ge\dfrac{ab}{c}+\dfrac{bc}{a}+\dfrac{ca}{b}\ge a+b+c\) (1)

Mà \(3\left(a+b+c\right)\ge\left(a^5+b^5+c^5\right)\left(a+b+c\right)\ge\left(a^3+b^3+c^3\right)^2\ge3\left(a^3b^3+b^3c^3+c^3a^3\right)\)

\(\Rightarrow a+b+c\ge a^3b^3+b^3c^3+c^3a^3\) (2)

Từ (1);(2) \(\Rightarrow\) đpcm

AH
Akai Haruma
Giáo viên
26 tháng 1 2021

Bạn tham khảo lời giải tại đây:

cho các số thực dưong x,y,z thỏa mãn : x2 y2 z2=3chứng minh rằng : \(\dfrac{x}{\sqrt[3]{yz}} \dfrac{y}{\sqrt[3]{zx}} \df... - Hoc24

AH
Akai Haruma
Giáo viên
26 tháng 1 2021

Cách khác:

Áp dụng BĐT AM-GM và BĐT Cauchy-Schwarz:

\(\sum \frac{x}{\sqrt[3]{yz}}\geq \sum \frac{x}{\frac{y+z+1}{3}}=3\sum \frac{x}{y+z+1}=3\sum \frac{x^2}{xy+xz+x}\)

\(\geq 3. \frac{(x+y+z)^2}{2(xy+yz+xz)+(x+y+z)}\)

Ta sẽ chứng minh: \(\frac{3(x+y+z)^2}{2(xy+yz+xz)+(x+y+z)}\geq xy+yz+xz(*)\)

Đặt $x+y+z=a$ thì $xy+yz+xz=\frac{a^2-3}{2}$

Bằng BĐT AM-GM dễ thấy $\sqrt{3}< a\leq 3$

BĐT $(*)$ trở thành:

$\frac{3a^2}{a^2+a-3}\geq \frac{a^2-3}{2}$

$\Leftrightarrow a^4+a^3-12a^2-3a+9\leq 0$

$\Leftrightarrow (a-3)(a+1)(a^2+3a-3)\leq 0$

Điều này đúng với mọi $\sqrt{3}< a\leq 3$

Do đó BĐT $(*)$ đúng nên ta có đpcm.

Dấu "=" xảy ra khi $x=y=z=1$

13 tháng 5 2017

\(xy+yz+zx-xyz=1-x-y-z+xy+yz+zx-xyz\)

\(=\left(1-x\right)-y\left(1-x\right)-z\left(1-x\right)+yz\left(1-x\right)\)

\(=\left(1-x\right)\left(1-y-z+yz\right)=\left(1-x\right)\left(1-y\right)\left(1-z\right)\)

\(xy+yz+zx+xyz+2=1+x+y+z+xy+yz+zx+xyz\)

\(=\left(1+x\right)+y\left(1+x\right)+z\left(1+x\right)+yz\left(1+x\right)\)

\(=\left(1+x\right)\left(1+y\right)\left(1+z\right)\)

\(1+x+y+z=1+1\Rightarrow1+x=\left(1-y\right)+\left(1-z\right)\ge2\sqrt{\left(1-y\right)\left(1-z\right)}\)

Tương tự ta cũng có: \(1+y\ge2\sqrt{\left(1-z\right)\left(1-x\right)}\)

\(1+z\ge2\sqrt{\left(1-x\right)\left(1-y\right)}\)

Vậy \(S\le\frac{\left(1-x\right)\left(1-y\right)\left(1-z\right)}{8\left(1-x\right)\left(1-y\right)\left(1-z\right)}=\frac{1}{8}\)

AH
Akai Haruma
Giáo viên
11 tháng 9 2023

Lời giải:
Áp dụng BĐT Cô-si:

$(x^2+y^2+z^2)(xy+yz+xz)^2=(x^2+y^2+z^2)(xy+yz+xz)(xy+yz+xz)$

$\leq \left(\frac{x^2+y^2+z^2+xy+yz+xz+xy+yz+xz}{3}\right)^3$

$=\frac{(x+y+z)^6}{27}=\frac{3^6}{27}=27$

Vậy max của biểu thức là $27$ khi $a=b=c=1$

10 tháng 5 2016

khó quá!!!!!!!!!!!